Find the closed form of $sum_{k=1}^{n}cos^{2m+1}left(frac{(2k-1)pi}{2n+1}right)$












1














Add it: Let $m,n$ be postive integers. Find the closed form of



$$f=sum_{k=1}^{n} cos^{2m+1}{left(dfrac{(2k-1)pi}{2n+1}right)}$$



for $m, n in mathbb{N}^{+}$.



Maybe use Euler



begin{align}
2cos{x} &=e^{ix}+e^{-ix} \
dfrac{pi}{2n+1} &=x
end{align}



then



begin{align}
f &= dfrac{1}{2^{2m+1}}sum_{k=1}^{n}(w^{(2k-1)}+w^{-(2k-1)})^{2m+1} \
&=dfrac{1}{2^{2m+1}}sum_{k=1}^{n}sum_{i=0}^{2m+1}binom{2m+1}{i}w^{-(2k-1)i}w^{((2m+1)-i)(2k-1)}\
&=dfrac{1}{(2w)^{2m+1}}sum_{i=0}^{2m+1}binom{2m+1}{i}w^{2i}sum_{k=1}^{n}w^{(4m-4i+2)k}
end{align}



where $w=e^{ix}$.










share|cite|improve this question




















  • 3




    What have you tried? You seem to have a demand rather than a question.
    – JimB
    Dec 26 '18 at 4:13










  • Last sum is geometric, isn't it?
    – saulspatz
    Dec 26 '18 at 8:00






  • 3




    It is misleading to use $i$ both as imaginary unit and a run index.
    – user
    Dec 26 '18 at 8:13












  • One simplification is to note that f=1/2 whenever $n>m$. Also, one could argue that what you present is already a "closed-form" solution
    – JimB
    Dec 27 '18 at 5:55












  • One of my previous questions is about a similar sum, and has a nice answer math.stackexchange.com/a/2395003/309055 which might help
    – MeMyselfI
    Jan 1 at 12:24
















1














Add it: Let $m,n$ be postive integers. Find the closed form of



$$f=sum_{k=1}^{n} cos^{2m+1}{left(dfrac{(2k-1)pi}{2n+1}right)}$$



for $m, n in mathbb{N}^{+}$.



Maybe use Euler



begin{align}
2cos{x} &=e^{ix}+e^{-ix} \
dfrac{pi}{2n+1} &=x
end{align}



then



begin{align}
f &= dfrac{1}{2^{2m+1}}sum_{k=1}^{n}(w^{(2k-1)}+w^{-(2k-1)})^{2m+1} \
&=dfrac{1}{2^{2m+1}}sum_{k=1}^{n}sum_{i=0}^{2m+1}binom{2m+1}{i}w^{-(2k-1)i}w^{((2m+1)-i)(2k-1)}\
&=dfrac{1}{(2w)^{2m+1}}sum_{i=0}^{2m+1}binom{2m+1}{i}w^{2i}sum_{k=1}^{n}w^{(4m-4i+2)k}
end{align}



where $w=e^{ix}$.










share|cite|improve this question




















  • 3




    What have you tried? You seem to have a demand rather than a question.
    – JimB
    Dec 26 '18 at 4:13










  • Last sum is geometric, isn't it?
    – saulspatz
    Dec 26 '18 at 8:00






  • 3




    It is misleading to use $i$ both as imaginary unit and a run index.
    – user
    Dec 26 '18 at 8:13












  • One simplification is to note that f=1/2 whenever $n>m$. Also, one could argue that what you present is already a "closed-form" solution
    – JimB
    Dec 27 '18 at 5:55












  • One of my previous questions is about a similar sum, and has a nice answer math.stackexchange.com/a/2395003/309055 which might help
    – MeMyselfI
    Jan 1 at 12:24














1












1








1


4





Add it: Let $m,n$ be postive integers. Find the closed form of



$$f=sum_{k=1}^{n} cos^{2m+1}{left(dfrac{(2k-1)pi}{2n+1}right)}$$



for $m, n in mathbb{N}^{+}$.



Maybe use Euler



begin{align}
2cos{x} &=e^{ix}+e^{-ix} \
dfrac{pi}{2n+1} &=x
end{align}



then



begin{align}
f &= dfrac{1}{2^{2m+1}}sum_{k=1}^{n}(w^{(2k-1)}+w^{-(2k-1)})^{2m+1} \
&=dfrac{1}{2^{2m+1}}sum_{k=1}^{n}sum_{i=0}^{2m+1}binom{2m+1}{i}w^{-(2k-1)i}w^{((2m+1)-i)(2k-1)}\
&=dfrac{1}{(2w)^{2m+1}}sum_{i=0}^{2m+1}binom{2m+1}{i}w^{2i}sum_{k=1}^{n}w^{(4m-4i+2)k}
end{align}



where $w=e^{ix}$.










share|cite|improve this question















Add it: Let $m,n$ be postive integers. Find the closed form of



$$f=sum_{k=1}^{n} cos^{2m+1}{left(dfrac{(2k-1)pi}{2n+1}right)}$$



for $m, n in mathbb{N}^{+}$.



Maybe use Euler



begin{align}
2cos{x} &=e^{ix}+e^{-ix} \
dfrac{pi}{2n+1} &=x
end{align}



then



begin{align}
f &= dfrac{1}{2^{2m+1}}sum_{k=1}^{n}(w^{(2k-1)}+w^{-(2k-1)})^{2m+1} \
&=dfrac{1}{2^{2m+1}}sum_{k=1}^{n}sum_{i=0}^{2m+1}binom{2m+1}{i}w^{-(2k-1)i}w^{((2m+1)-i)(2k-1)}\
&=dfrac{1}{(2w)^{2m+1}}sum_{i=0}^{2m+1}binom{2m+1}{i}w^{2i}sum_{k=1}^{n}w^{(4m-4i+2)k}
end{align}



where $w=e^{ix}$.







trigonometry






share|cite|improve this question















share|cite|improve this question













share|cite|improve this question




share|cite|improve this question








edited Dec 26 '18 at 7:53







inequality

















asked Dec 26 '18 at 4:04









inequalityinequality

701520




701520








  • 3




    What have you tried? You seem to have a demand rather than a question.
    – JimB
    Dec 26 '18 at 4:13










  • Last sum is geometric, isn't it?
    – saulspatz
    Dec 26 '18 at 8:00






  • 3




    It is misleading to use $i$ both as imaginary unit and a run index.
    – user
    Dec 26 '18 at 8:13












  • One simplification is to note that f=1/2 whenever $n>m$. Also, one could argue that what you present is already a "closed-form" solution
    – JimB
    Dec 27 '18 at 5:55












  • One of my previous questions is about a similar sum, and has a nice answer math.stackexchange.com/a/2395003/309055 which might help
    – MeMyselfI
    Jan 1 at 12:24














  • 3




    What have you tried? You seem to have a demand rather than a question.
    – JimB
    Dec 26 '18 at 4:13










  • Last sum is geometric, isn't it?
    – saulspatz
    Dec 26 '18 at 8:00






  • 3




    It is misleading to use $i$ both as imaginary unit and a run index.
    – user
    Dec 26 '18 at 8:13












  • One simplification is to note that f=1/2 whenever $n>m$. Also, one could argue that what you present is already a "closed-form" solution
    – JimB
    Dec 27 '18 at 5:55












  • One of my previous questions is about a similar sum, and has a nice answer math.stackexchange.com/a/2395003/309055 which might help
    – MeMyselfI
    Jan 1 at 12:24








3




3




What have you tried? You seem to have a demand rather than a question.
– JimB
Dec 26 '18 at 4:13




What have you tried? You seem to have a demand rather than a question.
– JimB
Dec 26 '18 at 4:13












Last sum is geometric, isn't it?
– saulspatz
Dec 26 '18 at 8:00




Last sum is geometric, isn't it?
– saulspatz
Dec 26 '18 at 8:00




3




3




It is misleading to use $i$ both as imaginary unit and a run index.
– user
Dec 26 '18 at 8:13






It is misleading to use $i$ both as imaginary unit and a run index.
– user
Dec 26 '18 at 8:13














One simplification is to note that f=1/2 whenever $n>m$. Also, one could argue that what you present is already a "closed-form" solution
– JimB
Dec 27 '18 at 5:55






One simplification is to note that f=1/2 whenever $n>m$. Also, one could argue that what you present is already a "closed-form" solution
– JimB
Dec 27 '18 at 5:55














One of my previous questions is about a similar sum, and has a nice answer math.stackexchange.com/a/2395003/309055 which might help
– MeMyselfI
Jan 1 at 12:24




One of my previous questions is about a similar sum, and has a nice answer math.stackexchange.com/a/2395003/309055 which might help
– MeMyselfI
Jan 1 at 12:24










1 Answer
1






active

oldest

votes


















1














The function $frac{(2n+1)/z}{z^{2n+1}-1}$ has residue $1$ at each $2n+1^text{st}$ root of unity. So we need to account for the residues at $0$ and $infty$.



$$newcommand{Res}{operatorname*{Res}}
begin{align}
sum_{k=1}^ncos^{2m+1}left(pi,frac{2k-1}{2n+1}right)
&=frac12-frac12sum_{k=0}^{2n}cos^{2m+1}left(2pi,frac{k}{2n+1}right)\
&=frac12+frac12Res_{z=0}left(frac1{2^{2m+1}}frac{(2n+1)/z}{z^{2n+1}-1}left(z+frac1zright)^{2m+1}right)\
&-frac12Res_{z=infty}left(frac1{2^{2m+1}}frac{(2n+1)/z}{z^{2n+1}-1}left(z+frac1zright)^{2m+1}right)\
&=frac12-frac{2n+1}{2^{2m+2}}left[z^0right]left(sum_{k=0}^infty z^{(2n+1)k}left(z+frac1zright)^{2m+1}right)\
&-frac{2n+1}{2^{2m+2}}left[z^0right]left(sum_{k=1}^infty z^{-(2n+1)k}left(z+frac1zright)^{2m+1}right)\
&=frac12-frac{2n+1}{2^{2m+1}}sum_{k=0}^mbinom{2m+1}{k}[,2n+1,|,2m+1-2k,]
end{align}
$$

where $[dots]$ are Iverson Brackets.



Note that for $mlt n$, the sum is $frac12$ because $2n+1$ cannot divide $2m+1-2k$.






share|cite|improve this answer























  • The original sum is finite, but contains cosines that are non-rational. This answer shows that the sum is rational and does not require the evaluation of any cosines.
    – robjohn
    yesterday











Your Answer





StackExchange.ifUsing("editor", function () {
return StackExchange.using("mathjaxEditing", function () {
StackExchange.MarkdownEditor.creationCallbacks.add(function (editor, postfix) {
StackExchange.mathjaxEditing.prepareWmdForMathJax(editor, postfix, [["$", "$"], ["\\(","\\)"]]);
});
});
}, "mathjax-editing");

StackExchange.ready(function() {
var channelOptions = {
tags: "".split(" "),
id: "69"
};
initTagRenderer("".split(" "), "".split(" "), channelOptions);

StackExchange.using("externalEditor", function() {
// Have to fire editor after snippets, if snippets enabled
if (StackExchange.settings.snippets.snippetsEnabled) {
StackExchange.using("snippets", function() {
createEditor();
});
}
else {
createEditor();
}
});

function createEditor() {
StackExchange.prepareEditor({
heartbeatType: 'answer',
autoActivateHeartbeat: false,
convertImagesToLinks: true,
noModals: true,
showLowRepImageUploadWarning: true,
reputationToPostImages: 10,
bindNavPrevention: true,
postfix: "",
imageUploader: {
brandingHtml: "Powered by u003ca class="icon-imgur-white" href="https://imgur.com/"u003eu003c/au003e",
contentPolicyHtml: "User contributions licensed under u003ca href="https://creativecommons.org/licenses/by-sa/3.0/"u003ecc by-sa 3.0 with attribution requiredu003c/au003e u003ca href="https://stackoverflow.com/legal/content-policy"u003e(content policy)u003c/au003e",
allowUrls: true
},
noCode: true, onDemand: true,
discardSelector: ".discard-answer"
,immediatelyShowMarkdownHelp:true
});


}
});














draft saved

draft discarded


















StackExchange.ready(
function () {
StackExchange.openid.initPostLogin('.new-post-login', 'https%3a%2f%2fmath.stackexchange.com%2fquestions%2f3052625%2ffind-the-closed-form-of-sum-k-1n-cos2m1-left-frac2k-1-pi2n1-r%23new-answer', 'question_page');
}
);

Post as a guest















Required, but never shown

























1 Answer
1






active

oldest

votes








1 Answer
1






active

oldest

votes









active

oldest

votes






active

oldest

votes









1














The function $frac{(2n+1)/z}{z^{2n+1}-1}$ has residue $1$ at each $2n+1^text{st}$ root of unity. So we need to account for the residues at $0$ and $infty$.



$$newcommand{Res}{operatorname*{Res}}
begin{align}
sum_{k=1}^ncos^{2m+1}left(pi,frac{2k-1}{2n+1}right)
&=frac12-frac12sum_{k=0}^{2n}cos^{2m+1}left(2pi,frac{k}{2n+1}right)\
&=frac12+frac12Res_{z=0}left(frac1{2^{2m+1}}frac{(2n+1)/z}{z^{2n+1}-1}left(z+frac1zright)^{2m+1}right)\
&-frac12Res_{z=infty}left(frac1{2^{2m+1}}frac{(2n+1)/z}{z^{2n+1}-1}left(z+frac1zright)^{2m+1}right)\
&=frac12-frac{2n+1}{2^{2m+2}}left[z^0right]left(sum_{k=0}^infty z^{(2n+1)k}left(z+frac1zright)^{2m+1}right)\
&-frac{2n+1}{2^{2m+2}}left[z^0right]left(sum_{k=1}^infty z^{-(2n+1)k}left(z+frac1zright)^{2m+1}right)\
&=frac12-frac{2n+1}{2^{2m+1}}sum_{k=0}^mbinom{2m+1}{k}[,2n+1,|,2m+1-2k,]
end{align}
$$

where $[dots]$ are Iverson Brackets.



Note that for $mlt n$, the sum is $frac12$ because $2n+1$ cannot divide $2m+1-2k$.






share|cite|improve this answer























  • The original sum is finite, but contains cosines that are non-rational. This answer shows that the sum is rational and does not require the evaluation of any cosines.
    – robjohn
    yesterday
















1














The function $frac{(2n+1)/z}{z^{2n+1}-1}$ has residue $1$ at each $2n+1^text{st}$ root of unity. So we need to account for the residues at $0$ and $infty$.



$$newcommand{Res}{operatorname*{Res}}
begin{align}
sum_{k=1}^ncos^{2m+1}left(pi,frac{2k-1}{2n+1}right)
&=frac12-frac12sum_{k=0}^{2n}cos^{2m+1}left(2pi,frac{k}{2n+1}right)\
&=frac12+frac12Res_{z=0}left(frac1{2^{2m+1}}frac{(2n+1)/z}{z^{2n+1}-1}left(z+frac1zright)^{2m+1}right)\
&-frac12Res_{z=infty}left(frac1{2^{2m+1}}frac{(2n+1)/z}{z^{2n+1}-1}left(z+frac1zright)^{2m+1}right)\
&=frac12-frac{2n+1}{2^{2m+2}}left[z^0right]left(sum_{k=0}^infty z^{(2n+1)k}left(z+frac1zright)^{2m+1}right)\
&-frac{2n+1}{2^{2m+2}}left[z^0right]left(sum_{k=1}^infty z^{-(2n+1)k}left(z+frac1zright)^{2m+1}right)\
&=frac12-frac{2n+1}{2^{2m+1}}sum_{k=0}^mbinom{2m+1}{k}[,2n+1,|,2m+1-2k,]
end{align}
$$

where $[dots]$ are Iverson Brackets.



Note that for $mlt n$, the sum is $frac12$ because $2n+1$ cannot divide $2m+1-2k$.






share|cite|improve this answer























  • The original sum is finite, but contains cosines that are non-rational. This answer shows that the sum is rational and does not require the evaluation of any cosines.
    – robjohn
    yesterday














1












1








1






The function $frac{(2n+1)/z}{z^{2n+1}-1}$ has residue $1$ at each $2n+1^text{st}$ root of unity. So we need to account for the residues at $0$ and $infty$.



$$newcommand{Res}{operatorname*{Res}}
begin{align}
sum_{k=1}^ncos^{2m+1}left(pi,frac{2k-1}{2n+1}right)
&=frac12-frac12sum_{k=0}^{2n}cos^{2m+1}left(2pi,frac{k}{2n+1}right)\
&=frac12+frac12Res_{z=0}left(frac1{2^{2m+1}}frac{(2n+1)/z}{z^{2n+1}-1}left(z+frac1zright)^{2m+1}right)\
&-frac12Res_{z=infty}left(frac1{2^{2m+1}}frac{(2n+1)/z}{z^{2n+1}-1}left(z+frac1zright)^{2m+1}right)\
&=frac12-frac{2n+1}{2^{2m+2}}left[z^0right]left(sum_{k=0}^infty z^{(2n+1)k}left(z+frac1zright)^{2m+1}right)\
&-frac{2n+1}{2^{2m+2}}left[z^0right]left(sum_{k=1}^infty z^{-(2n+1)k}left(z+frac1zright)^{2m+1}right)\
&=frac12-frac{2n+1}{2^{2m+1}}sum_{k=0}^mbinom{2m+1}{k}[,2n+1,|,2m+1-2k,]
end{align}
$$

where $[dots]$ are Iverson Brackets.



Note that for $mlt n$, the sum is $frac12$ because $2n+1$ cannot divide $2m+1-2k$.






share|cite|improve this answer














The function $frac{(2n+1)/z}{z^{2n+1}-1}$ has residue $1$ at each $2n+1^text{st}$ root of unity. So we need to account for the residues at $0$ and $infty$.



$$newcommand{Res}{operatorname*{Res}}
begin{align}
sum_{k=1}^ncos^{2m+1}left(pi,frac{2k-1}{2n+1}right)
&=frac12-frac12sum_{k=0}^{2n}cos^{2m+1}left(2pi,frac{k}{2n+1}right)\
&=frac12+frac12Res_{z=0}left(frac1{2^{2m+1}}frac{(2n+1)/z}{z^{2n+1}-1}left(z+frac1zright)^{2m+1}right)\
&-frac12Res_{z=infty}left(frac1{2^{2m+1}}frac{(2n+1)/z}{z^{2n+1}-1}left(z+frac1zright)^{2m+1}right)\
&=frac12-frac{2n+1}{2^{2m+2}}left[z^0right]left(sum_{k=0}^infty z^{(2n+1)k}left(z+frac1zright)^{2m+1}right)\
&-frac{2n+1}{2^{2m+2}}left[z^0right]left(sum_{k=1}^infty z^{-(2n+1)k}left(z+frac1zright)^{2m+1}right)\
&=frac12-frac{2n+1}{2^{2m+1}}sum_{k=0}^mbinom{2m+1}{k}[,2n+1,|,2m+1-2k,]
end{align}
$$

where $[dots]$ are Iverson Brackets.



Note that for $mlt n$, the sum is $frac12$ because $2n+1$ cannot divide $2m+1-2k$.







share|cite|improve this answer














share|cite|improve this answer



share|cite|improve this answer








edited yesterday

























answered yesterday









robjohnrobjohn

265k27303625




265k27303625












  • The original sum is finite, but contains cosines that are non-rational. This answer shows that the sum is rational and does not require the evaluation of any cosines.
    – robjohn
    yesterday


















  • The original sum is finite, but contains cosines that are non-rational. This answer shows that the sum is rational and does not require the evaluation of any cosines.
    – robjohn
    yesterday
















The original sum is finite, but contains cosines that are non-rational. This answer shows that the sum is rational and does not require the evaluation of any cosines.
– robjohn
yesterday




The original sum is finite, but contains cosines that are non-rational. This answer shows that the sum is rational and does not require the evaluation of any cosines.
– robjohn
yesterday


















draft saved

draft discarded




















































Thanks for contributing an answer to Mathematics Stack Exchange!


  • Please be sure to answer the question. Provide details and share your research!

But avoid



  • Asking for help, clarification, or responding to other answers.

  • Making statements based on opinion; back them up with references or personal experience.


Use MathJax to format equations. MathJax reference.


To learn more, see our tips on writing great answers.





Some of your past answers have not been well-received, and you're in danger of being blocked from answering.


Please pay close attention to the following guidance:


  • Please be sure to answer the question. Provide details and share your research!

But avoid



  • Asking for help, clarification, or responding to other answers.

  • Making statements based on opinion; back them up with references or personal experience.


To learn more, see our tips on writing great answers.




draft saved


draft discarded














StackExchange.ready(
function () {
StackExchange.openid.initPostLogin('.new-post-login', 'https%3a%2f%2fmath.stackexchange.com%2fquestions%2f3052625%2ffind-the-closed-form-of-sum-k-1n-cos2m1-left-frac2k-1-pi2n1-r%23new-answer', 'question_page');
}
);

Post as a guest















Required, but never shown





















































Required, but never shown














Required, but never shown












Required, but never shown







Required, but never shown

































Required, but never shown














Required, but never shown












Required, but never shown







Required, but never shown







Popular posts from this blog

Mario Kart Wii

What does “Dominus providebit” mean?

Antonio Litta Visconti Arese